*** empty log message ***
This commit is contained in:
parent
c435621015
commit
2f7dae0b35
1 changed files with 139 additions and 0 deletions
139
194.tex
Normal file
139
194.tex
Normal file
|
@ -0,0 +1,139 @@
|
|||
\magnification=1200
|
||||
|
||||
\noindent
|
||||
{\it George Weigt --- Advanced Calculus Homework \#10}
|
||||
|
||||
\beginsection Page 312, problem 1a.
|
||||
|
||||
Evaluate the line integral
|
||||
$$\int_C z\,dx+x\,dy+y\,dz$$
|
||||
where $C$ is the curve that for $0\le t\le 2\pi$ is parameterized as
|
||||
$$\eqalign{
|
||||
x&=\cos t\cr
|
||||
y&=\sin t\cr
|
||||
z&=t\cr
|
||||
}$$
|
||||
|
||||
\bigskip
|
||||
\noindent
|
||||
Solution: We have
|
||||
$$\eqalign{
|
||||
dx&=-\sin t\,dt\cr
|
||||
dy&=\cos t\,dt\cr
|
||||
dz&=dt\cr
|
||||
}$$
|
||||
Hence
|
||||
$$\eqalign{
|
||||
\int_C z\,dx+x\,dy+y\,dz
|
||||
&=\int_0^{2\pi}\left(-t\sin t+\cos^2 t+\sin t\right)dt\cr
|
||||
&=\left(t\cos t-\sin t+{t\over2}+{\sin t\cos t\over2}-\cos t\right)\bigg|_0^{2\pi}\cr
|
||||
&=(2\pi-0+\pi+0-1)-(0-0+0+0-1)\cr
|
||||
&=3\pi\cr
|
||||
}$$
|
||||
|
||||
\beginsection Page 312, problem 1b.
|
||||
|
||||
Evaluate the line integral
|
||||
$$\int_C x^2\,dx-xz\,dy+y^2\,dz$$
|
||||
where $C$ is a straight line form $(1,0,1)$ to $(2,3,2)$.
|
||||
|
||||
\bigskip
|
||||
\noindent
|
||||
Solution: Let $0\le t\le 1$.
|
||||
Then $C$ can be parameterized as follows.
|
||||
$$\eqalign{
|
||||
x&=1+t\cr
|
||||
y&=3t\cr
|
||||
z&=1+t\cr
|
||||
}$$
|
||||
We have
|
||||
$$dx=dt,\qquad dy=3\,dt,\qquad dz=dt$$
|
||||
Hence
|
||||
$$\eqalign{
|
||||
\int_C x^2\,dx-xz\,dy+y^2\,dz
|
||||
&=\int_0^1[(t+1)^2-3(t+1)^2+9t^2]\,dt\cr
|
||||
&=\int_0^1(7t^2-4t-2)\,dt\cr
|
||||
&=\left({7t^3\over3}-2t^2-2t\right)\bigg|_0^1\cr
|
||||
&=-{5\over3}\cr
|
||||
}$$
|
||||
|
||||
\beginsection Page 312, problem 1d.
|
||||
|
||||
Evaluate the line integral
|
||||
$$\int_C u_T\,ds$$
|
||||
where ${\bf u}=2xy^2z{\bf i}+2x^2yz{\bf j}+x^2y^2{\bf k}$,
|
||||
and $C$ is the circle $x=\cos t$, $y=\sin t$, $z=2$, directed by increasing $t$.
|
||||
|
||||
\bigskip
|
||||
\noindent
|
||||
Solution: We have
|
||||
$$dx=-\sin t\,dt,\qquad dy=\cos t\,dt,\qquad dz=0$$
|
||||
hence
|
||||
$$\eqalign{
|
||||
I&=\int_C 2xy^2z\,dx+2x^2yz\,dy+x^2y^2\,dz\cr
|
||||
&=\int_0^{2\pi} 4(\cos t)(\sin^2 t)(-\sin t)\,dt+4(\cos^2 t)(\sin t)(\cos t)\,dt\cr
|
||||
&=4\int_0^{2\pi}(-\cos t\sin^3 t+\cos^3 t\sin t)\,dt\cr
|
||||
&=4\int_0^{2\pi}\sin t\cos t\,(-\sin^2 t+\cos^2 t)\,dt\cr
|
||||
}$$
|
||||
Now for some trigonometric gymnastics. We have
|
||||
$$\sin t\cos t={\sin2t\over 2}\qquad\cos^2t-\sin^2t=\cos 2t$$
|
||||
hence
|
||||
$$4\sin t\cos t\,(-\sin^2 t+\cos^2 t)=2\sin2t\cos2t
|
||||
=\sin4t$$
|
||||
Therefore
|
||||
$$I=\int_0^{2\pi}\sin4t\,dt=-{\cos 4t\over4}\bigg|_0^{2\pi}=0$$
|
||||
|
||||
\beginsection Page 312, problem 1e.
|
||||
|
||||
Evaluate the line integral
|
||||
$$\int_C u_T\,ds$$
|
||||
where ${\bf u}=\mathop{\rm curl}{\bf v}$, ${\bf v}=y^2{\bf i}+z^2{\bf j}+x^2{\bf k}$,
|
||||
and $C$ is the path $x=2t+1$, $y=t^2$, $z=1+t^3$, $0\le t\le 1$, directed by increasing $t$.
|
||||
|
||||
\bigskip
|
||||
\noindent
|
||||
Solution: We have
|
||||
$$\mathop{\rm curl}{\bf v}
|
||||
=\left(\matrix{
|
||||
\partial x^2/\partial y-\partial z^2/\partial z\cr
|
||||
\partial y^2/\partial z-\partial x^2/\partial x\cr
|
||||
\partial z^2/\partial x-\partial y^2/\partial y\cr
|
||||
}\right)=\left(\matrix{-2z\cr-2x\cr-2y}\right)
|
||||
$$
|
||||
Therefore
|
||||
$$\int_C u_T\,ds=-2\int_C z\,dx+x\,dy+y\,dz$$
|
||||
From the given parameteric functions we have
|
||||
$$dx=2\,dt,\qquad dy=2t\,dt,\qquad dz=3t^2\,dt$$
|
||||
Hence
|
||||
$$\eqalign{
|
||||
\int_C u_T\,ds
|
||||
&=-2\int_0^1 (1+t^3)(2\,dt)+(2t+1)(2t\,dt)+3t^4\,dt\cr
|
||||
&=-2\int_0^1 (3t^4+2t^3+4t^2+2t+2)\,dt\cr
|
||||
&=-2\left({3t^5\over5}+{t^4\over2}+{4t^3\over3}+t^2+2t\right)\bigg|_0^1\cr
|
||||
&=-{163\over15}\cr
|
||||
}$$
|
||||
|
||||
\beginsection Green's Theorem Strikes Back!
|
||||
|
||||
We have
|
||||
$$P=2y-3x+5+e^y(y-1),\qquad Q=4x-3y-2+xye^y$$
|
||||
and
|
||||
$${\partial P\over\partial y}=ye^y+2,\qquad{\partial Q\over\partial x}=ye^y+4$$
|
||||
Let $C_{12}$ be the path $C$ followed by the path from $(-1,0)$ back to the
|
||||
starting point $(1,0)$ so that $C_{12}$ is
|
||||
a closed loop.
|
||||
By Green's Theorem we have
|
||||
$$\int_{C_{12}} P\,dx+Q\,dy=\int\!\!\!\int_R
|
||||
\left({\partial Q\over\partial x}-{\partial P\over\partial y}\right)\,dx\,dy
|
||||
=2\int\!\!\!\int_R dx\,dy=2$$
|
||||
Now we have to subtract out the line integral over the path that we added to make a closed loop.
|
||||
Let $C_2$ be the path from $(-1,0)$ to $(1,0)$. In addition, let
|
||||
$$x=t,\qquad y=0,\qquad dx=dt,\qquad dy=0,\qquad-1\le t\le 1$$
|
||||
Then
|
||||
$$\int_{C_2} P\,dx+Q\,dy=\int_{-1}^1 (-3t+4)\,dt
|
||||
=\left(-{3t^2\over2}+4t\right)\bigg|_{-1}^1=8$$
|
||||
Therefore the final answer is
|
||||
$$\int_C=\int_{C_{12}}-\int_{C_2}=2-8=-6$$
|
||||
|
||||
|
||||
\end
|
Loading…
Add table
Add a link
Reference in a new issue